Which statements correctly compare the sum and product? Check all that apply. 1. Both functions have a constant rate of change. 2. The sum is linear and the product is quadratic. 3. The x-intercepts are the same. 4. The sum has one y-intercept and the product has two x-intercepts. 5. Both functions decrease for x > 2

Answers

Answer 1

Answer:

Step-by-step explanation:

B) the sum is linear and the product is quadratic

D) the sum has one y-intercept and the product has two x-intercepts

Answer 2

The sum is linear and the product is quadratic and The sum has one y-intercept and the product has two x-intercepts are two statements which compare the sum and product

What is Equation?

Two or more expressions with an Equal sign is called as Equation.

We need to find the statements which correctly compare the sum and product

The given statements are

1. Both functions have a constant rate of change.

2. The sum is linear and the product is quadratic.

3. The x-intercepts are the same.

4. The sum has one y-intercept and the product has two x-intercepts.

5. Both functions decrease for x > 2

A linear function simply means a function that represents a straight line on the coordinate plane.

In this case, the sum is linear and the product is quadratic while the sum has one y-intercept and the product has two x-intercepts.

Hence, the sum is linear and the product is quadratic and The sum has one y-intercept and the product has two x-intercepts are two statements which compare the sum and product

To learn more on Equation:

https://brainly.com/question/10413253

#SPJ2


Related Questions

The segment with endpoints (-1,4) & (2,8) has a distance of

Answers

Answer:

5

Step-by-step explanation:

(X1,Y1) = (-1,4)

(X2,Y2) = (2,8)

Solve the following equation: 3x - 7 = 9 + 2x.
A)12
B)16
c)20
d)24

Answers

answer:
B

the equation:
3x-7=9+2x

step 1: subtract 2x from both sides
x-7=9

step 2: add 7 to both sides
x=16

Answer:

Option B

Step-by-step explanation:

[tex]3x - 7 = 9 + 2x\\\\3x-7+7=9+2x+7\\\\3x = 9 + 7 + 2x\\\\3x=16+2x\\\\3x-2x=16+2x-2x\\\\\boxed{x=16}[/tex]

Hope this helps!

Evaluate f (x) = 2x – 13 for f (x) = 9, x =

Answers

Answer:

11

Step-by-step explanation:

f(x) = 2x - 13

9 = 2x - 13                   Add 13 to both sides

9 + 13 = 2x                  Add the left

22 = 2x                       Divide by 2

22/2 = x

x = 11

What is the average density of a laptop?

Answers

Throughout all laptops and desktops what is the standard pixels per inch (PPI) used? Does it vary throughout screen ratio? I found mine to be 100ppi



Hope that helps

Solve for x 9x - 1 = -11

Answers

Answer:

x=[tex]-\frac{10}{9}[/tex]

Step-by-step explanation:

We start by moving the constant to the right. In this case -1 (it becomes positive 1).

9x=-11 +1

9x= - 10

divide 9 by both sides.

x=- [tex]\frac{10}{9}[/tex]

solve this equation
3x+6=-2-2x

Answers

Answer:

x = -8/5

x = -1.6

Step-by-step explanation:

3x+6=-2-2x

Add 2x to each side

3x+2x+6=-2-2x+2x

5x+6 = -2x

Subtract 6 from each side

5x+6-6 = -2-6

5x= -8

Divide each side by 5

5x/5 = -8/5

x = -8/5

x = -1.6

What is the sum?

−1.5+1.9

Enter your answer, as a decimal, in the box.

Answers

Answer:

0.4

Step-by-step explanation:

Answer: it's 0.4 as a decimal

Step-by-step explanation:

K(h-j)+10’ if h= 7, j=-8, and k=5

Answers

Answer:

49

Step-by-step explanation:

5(7-8)+10

49

it's equals 49

PLEASEEE HELPPP

Solve for x:

Answers

Second option, just do Pemdas backwards.

Help please TYSM IF YOU DO!

Answers

Answer:

I think it is mean

Step-by-step explanation:

because mean would be the average of the scores

Answer: The mean

Step-by-step explanation:

The mean will show the average of the teams scores.

-5/6 + 1/4 = is what???
Please help th points is 100...

Answers

Answer:

-7/12

Step-by-step explanation:

Find common denominators. Note that what you do to the denominator, you do to the numerator.

Set the common denominator to 12:

(-5/6)(2/2) = -10/12

(1/4)(3/3) = 3/12

Combine the terms:

-10/12 + 3/12 = 3/12 - 10/12 = -7/12

-7/12 is your answer.

~

the sum of two numbers is 58. The larger number is 22 more than the smaller number. What are the numbers?

Answers

the two numbers are 40 and 18

Answer to this PLEASE GOD BLESS YOU

Answers

Answer:

the answer to this question is b

Write the next 4 digits in the repeating decimal 4.715

Answers

Answer:

4.7157157

Step-by-step explanation:

because it is a repeating decimal, it will repeat the terms that come first such as 715.

Nathan is stocking bathrooms at the hotel where he works. he has 18 rolls of toilet paper and 9 bars of soap. if he wants all bathrooms to be stocked identically, with the same combination of supplies in each one and nothing left over, what is the greatest combination of bathrooms Nathan can stock

Answers

I would say 3 because 18/3 is 6, and 9/3 is 3. So you could put 6 rolls of toilet paper in each bathroom and three bars of soap. I probably just got this wrong but hope I helped out :)

5. Solve the following equations for the indicated variable.
a. 5(3x + 7)=20-2(x +1) for x
b. 2x - y= 24 for y
c. m(3 – 4m)= 7+4(8 – m2) for m
d. 5x(x + 3) =(x)(5x – 3) + 36 for x

Answers

Answer:a) x =−1

B) x=1/2y+12

C) m=13

D) x=2

Step-by-step explanation:a) Let's solve your equation step-by-step.

5(3x+7)=20−2(x+1)

Step 1: Simplify both sides of the equation.

5(3x+7)=20−2(x+1)

(5)(3x)+(5)(7)=20+(−2)(x)+(−2)(1)(Distribute)

15x+35=20+−2x+−2

15x+35=(−2x)+(20+−2)(Combine Like Terms)

15x+35=−2x+18

15x+35=−2x+18

Step 2: Add 2x to both sides.

15x+35+2x=−2x+18+2x

17x+35=18

Step 3: Subtract 35 from both sides.

17x+35−35=18−35

17x=−17

Step 4: Divide both sides by 17.

17x/17=−17/17

x=−1

B) Let's solve for x.

2x−y=24

Step 1: Add y to both sides.

2x−y+y=24+y

2x=y+24

Step 2: Divide both sides by 2.

2x/2=y+24/2

x=1/2y+12

C) Let's solve your equation step-by-step.

m(3−4m)=7+4(8−m2)

Step 1: Simplify both sides of the equation.

−4m2+3m=−4m2+39

Step 2: Add 4m^2 to both sides.

−4m2+3m+4m2=−4m2+39+4m2

3m=39

Step 3: Divide both sides by 3.

3m/3=39/3

m=13

D) Let's solve your equation step-by-step.

5x(x+3)=x(5x−3)+36

Step 1: Simplify both sides of the equation.

5x2+15x=5x2−3x+36

Step 2: Subtract 5x^2 from both sides.

5x2+15x−5x2=5x2−3x+36−5x2

15x=−3x+36

Step 3: Add 3x to both sides.

15x+3x=−3x+36+3x

18x=36

Step 4: Divide both sides by 18.

18x/18=36/18

x=2

2/5 divided 1/2 as a unit rate

Answers

Answer:

2/5 divided 1/2 as a unit rate .what do you mean

2/5 divided 1/2 is 3/7 as a unit rate

How do you write.09 in words?

Answers

Answer:

zero point 9

0 and nine tenths

Step-by-step explanation:

G(n) = n^2 +4n find g(2)

Answers

Answer:

12

Step-by-step explanation:

G(2)= 2^2 +4(2)

Solve via order of operations

G(2)=4 +8

G(2)=12

108 ÷ 9 x (-12) ÷ 6 - (100 ÷ 5)

Answers

Answer:

-44

Step-by-step explanation:

108 ÷ 9 x (-12) ÷ 6 - (100 ÷ 5)

= 108 ÷ 9 x (-12) ÷ 6 - (20)

= 12 x (-12) ÷ 6 - (20)

= -144÷ 6 - (20)

= -24 - (20)

= -44

34+987=what please help

Answers

Answer:

1021

Step-by-step explanation:

i added 34 +987 and got 1021

1021 because 34+987= 1021

During a sale, 20-cent candy bars were sold at 3 for 50 cents. How much is saved on 9 bars?

Answers

Answer:

Step-by-step explanation:

$0.30

Step-by-step explanation:

1 bar of candy = $0.20

3 bars of candy = $0.50

To solve, multiply for both:

If you pay for each candy bar individually, they each cost $0.20. Multiply 9 with 0.20:

9 x 0.20 = $1.80

If you pay for the candy bars by 3's, they cost $0.50 each pack. Divide 9 with 3, then multiply by 0.50:

9/3 = 3

3 x 0.50 = $1.50

Subtract the total cost of the individual from the pack:

$1.80 - $1.50 = $0.30

. $0.30 is your answer.


The amount of money invested in a retirement fund is an example of which of the following?
a. investment asset
b. liquid asset
c. long term asset
d. use asset

answer: investment asset e2020

Answers

Answer:

A.

Step-by-step explanation:

Retirement funds are not liquid assets. You fund them with investment assets.  However, Roth IRA's and 401 (k) are not assets in the traditional sense. It may be confusing, but the way they grow is by investments. The answer is A.

The amount of money invested in a retirement fund is an example of an investment asset. which is the correct answer would be an option (A)

What is an Investment?

An Investment is defined as an asset or object purchased with the intention of generating income or appreciation is referred to as an investment. The term "appreciation" describes a rise in an asset's worth over time. When a person buys a product as an investment, they don't intend to utilize it right away; instead, they plan to use it to make money later on.

An investment always entails the expenditure of some capital today—time, effort, money, or an asset in the anticipation of receiving a larger return than what was first invested.

Investment portfolios are not movable property. They are financed by investments. One type of investing asset is the amount of money invested into a retirement fund.

Hence, the amount of money invested in a retirement fund is an example of an investment asset. which is the correct answer would be an option (A)

Learn more about an Investment asset here:

https://brainly.com/question/23611837

#SPJ2

Simplify: 5m (5m* +5m -4)
A 10m +10m² + m
B 10m + 10m + m
25m + 25m-20m
D 25m + 25m-20m

Answers

C I think because if you multiply 5 to everything you would get c

Seven is part of all of the following sets of numbers except
O irrational numbers
O integers
O rational numbers
natural numbers

Answers

Answer:

Seven isn't part of the set of irrational numbers.

Step-by-step explanation:

Seven and the Set of all Natural Numbers

Start with the smallest set among the choices. The set of all natural numbers, [tex]\mathbb{N}[/tex], starts with [tex]0[/tex] (or [tex]1[/tex], for some people.) A number [tex]n[/tex] is in [tex]\mathbb{N}\!\!\![/tex] (write [tex]n \in \mathbb{N}[/tex]) if and only if [tex](n - 1)[/tex] is in [tex]\mathbb{N}\![/tex]. Conversely, if [tex]n\![/tex] is indeed in [tex]\mathbb{N}\!\!\!\!\![/tex], then [tex](n + 1)[/tex] would also be in [tex]\mathbb{N}\!\!\!\!\!\!\![/tex]. For [tex]7[/tex]:

[tex]1 \in \mathbb{N} \implies 2 \in \mathbb{N}[/tex].

[tex]\vdots[/tex].

[tex]6 \in \mathbb{N} \implies 7 \in \mathbb{N}[/tex].

Therefore, [tex]7[/tex] is indeed in the set of all natural numbers.

Seven and the Set of all Integers

Similarly, a number [tex]n[/tex] is in the set of integers, [tex]\mathbb{Z}[/tex], if and only if either [tex](n - 1)[/tex] or [tex](n + 1)[/tex] is (or both are) in [tex]\mathbb{Z}\!\![/tex].

Conversely, if a number [tex]n[/tex] is in [tex]\mathbb{Z}[/tex], then both [tex](n - 1)[/tex] and [tex](n + 1)[/tex] will be in [tex]\mathbb{Z}\![/tex].

It can be shown in a similar iterative way that [tex]7 \in \mathbb{Z}[/tex].

Alternatively, consider the fact that the set of all natural numbers, [tex]\mathbb{N}[/tex], is a subset of the set of all integers, [tex]\mathbb{Z}[/tex]. Therefore, [tex]7 \in \mathbb{N}[/tex] implies that [tex]7 \in \mathbb{Z}[/tex].

Seven and the Set of all Rational Numbers

A number [tex]m[/tex] is a member of the set of all rational numbers [tex]\mathbb{Q}[/tex] if and only if there exists two integers [tex]p[/tex] and [tex]q[/tex] such that:

[tex]\displaystyle m = \frac{p}{q}[/tex].

[tex]1[/tex] and [tex]7[/tex] are both integers. If [tex]p = 7[/tex] and [tex]q = 1[/tex], then [tex]\displaystyle 7 = \frac{7}{1} = \frac{p}{q}[/tex]. Hence,

Alternatively, note that the set of all integers, [tex]\mathbb{Z}[/tex], is a subset of the set of all rational numbers, [tex]\mathbb{Q}[/tex]. Therefore, the fact that [tex]7 \in \mathbb{Z}[/tex] would imply that [tex]7 \in \mathbb{Q}[/tex].

Seven and the Set of all Irrational Numbers

A number is in the set of all irrational numbers if and only if:

this number is in the set of all real numbers, andthis number is not in the set of all rational numbers. (Hence "irrational.")

Therefore, the fact that [tex]7[/tex] is a rational number implies that it is not an irrational number.

Answer:

irrational numbers

Step-by-step explanation:

is 1 1/2 a rational number?

Answers

yes. It would be an irrational number if it didn't repeat in a pattern.

hope it helps comment if u have any questions

Answer:

Yes.

Step-by-step explanation:

Yes 1 [tex]\frac{1}{2}[/tex] is a rational number because because it can be converted to a decimal, which is 1.5

Solve the equation for y

Answers

the answer is C). y= 1/4x + 2

Round 10.999244792948 to the nearest whole number

Answers

Answer:

11

Step-by-step explanation:

1. Listing Information

The number is 10.999244792948.

Simply put, numbers below 5 are rounded down and numbers that are greater than or equal to five are rounded up.

The tenths place value in 10.999244792948 is 9.

2. Solving the Problem

With the previous information in mind, 9 is greater than 5. Because of this, 10.999244792948 should be rounded to 11.

Please answerrr I need help with hw and I still can't solve this!!!

Answers

The answer would be 16 all you have to do is multiply the length by the width which is 2 and 8.

If 4 times a number is 156, then 1/3 of the number is

Answers

4 divided by 156 is 39. 39 divided by 1/3 is 13

Answer:

[tex]\huge\boxed{\sf \frac{x}{3} = 12}[/tex]

Step-by-step explanation:

Let the number be x

4x = 156

Dividing both sides by 4

x = 36

Dividing both sides by 3

[tex]\frac{x}{3}= \frac{36}{3}[/tex]

[tex]\frac{x}{3} = 12[/tex]

Other Questions
What is the Veteran Administrations primary role in the mortgage market?A) To insure loans made by approved lenders.B) To guarantee loans made by lenders.C) To lend money directly to borrowers.D) To buy loans from other secondary mortgage marketers. There are 360 degrees in a circle. How many arc seconds will be in 96 % of a circle? The perimeter of a rectangle is 16cm. If the length is doubled, the perimeter is 26. What are the length and width of the rectangle? Please explain all steps required, thanks ahead of time! I need the answer please SOMEBODY HELP ASAP!! 33 POINTS! Which of the following are observations? Select all that apply.A. Watching table salt dissolve in water.B. Feeling the sensation of ice held in the hand.C. Smelling ammonia used in a cleaning solution.D. Hearing a loud bang when a balloon filled with hydrogen gas is ignited.E. Looking at the readout of a digital scale when measuring reagents for an experiment. Algebra 2 Help! Please help. dilations help me please How does herd behavior help explain how and why Ponyboy might have become a Greaser? what are the steps involved in the scientific method Can you help me i will give you a branlist What are the components of vector a Help I cant find the answer!! What is the surface area, in square inches, of a prism with a length of 12 inches, a width of 9 inches, and a height of 2 inches? pls help i will mark brainliest......i have a time limit GEOMETRY HELP PLEASE, ITS URGENT As part of their application for a loan to buy Lakeside Farm, a property they hope to develop as a bed-and-breakfast operation, the prospective owners have projected:Monthly fixed cost (loan payment, taxes, insurance, maintenance) $6000Variable cost per occupied room per night $ 20Revenue per occupied room per night $ 75a. Write the expression for total cost per month. Assume 30 days per month.b. Write the expression for total revenue per month, first using the general variables, and second including the values of known variables).c. If there are 12 guest rooms available, can they break even? What would be the profit with 12 rooms?d. What percentage of rooms would need to be occupied, on average, to break even? How might monarchs react to the ideas of the Enlightenment? By signing the document as they did, the signers (John Hancock, Thomas Jefferson, John Adams, etc.)became criminals. Which crime did they commit when they signed the Declaration of Independence? T - (desayunar) en un caf pequeo en el pueblo.